LSAT and Law School Admissions Forum

Get expert LSAT preparation and law school admissions advice from PowerScore Test Preparation.

User avatar
 Dave Killoran
PowerScore Staff
  • PowerScore Staff
  • Posts: 5852
  • Joined: Mar 25, 2011
|
#59775
Complete Question Explanation
(The complete setup for this game can be found here: lsat/viewtopic.php?t=26553)

The correct answer choice is (B)

If V is assigned to bench 2, then, by application of the super-block, F must be assigned to bench 2, and J must be assigned to bench 3. Then, when W is assigned to bench 4, this affects G, who can no longer be assigned to bench 4, or benches 2 and 3 since they are taken by F and J. Therefore, G must be assigned to bench 1:

pt26_s98_g1_q2.png
Since G must be assigned to bench 1, answer choice (B) is proven correct. Additionally, when G is assigned to bench 1, H is forced into bench 4. The last two variables, X and Y, create a dual option that rotates between benches 1 and 3.
You do not have the required permissions to view the files attached to this post.

Get the most out of your LSAT Prep Plus subscription.

Analyze and track your performance with our Testing and Analytics Package.